Class 11 Physics | Rigid Body Dynamics | #25 Example-10 on Rotational Motion | For JEE & NEET

  Рет қаралды 45,853

Physics Galaxy

Physics Galaxy

Күн бұрын

Пікірлер: 64
@alphaiitd1
@alphaiitd1 3 жыл бұрын
Thanks sir
@3511kat
@3511kat 11 жыл бұрын
Your solved example has helped me to solve a problem about angular momentum in my assignment. Thank you very much!
@Rajatkumar-dm5uf
@Rajatkumar-dm5uf 8 жыл бұрын
sir didn't understand why angular momenetum would be conserved about the hinge point ?
@harsha8412
@harsha8412 7 жыл бұрын
torque = force x perpendicular distance from axis of rotation. As distance is zero, torque is zero and L is conserved
@tinkywinky9099
@tinkywinky9099 7 жыл бұрын
Harsha Vardhan no, the force is 0 hence its conserved. as from COM distance to point A is not 0 .
@HarshitJangra-z5v
@HarshitJangra-z5v Ай бұрын
Why force is zero
@ayushsaxena1164
@ayushsaxena1164 6 жыл бұрын
Great deal of knowledge !! Appreciated!!!!!😀😀😀🙄🙄
@tpsicmin
@tpsicmin 3 жыл бұрын
Thank You Sir😎
@harsh17_17
@harsh17_17 6 жыл бұрын
Sir if i do with linear momentum Mu/2=3Mv/2 ,,,,v=u/3 ,,,then v=rw ,,u/3l ???? Aa gya sir plz clear it
@Mr.Tapori
@Mr.Tapori 7 ай бұрын
Same doubt
@harrymecwan3426
@harrymecwan3426 9 жыл бұрын
what about net force acting on it ... is it zero? if it is, then momentum must also be conserved..
@shivamverma9447
@shivamverma9447 4 жыл бұрын
Hinge forces are present
@sanchitmehta7181
@sanchitmehta7181 3 жыл бұрын
can we use energy conservation here ?
@JUSTFUN-wm8mf
@JUSTFUN-wm8mf 5 жыл бұрын
Sir if we think this question as like mine modified question ......The rod is moving rotationally and after that the ball strikes so can we use the intital angular momentum as ML^2w0/3 (wo is initial angular speed) because of rod ang. Momentum and the final ang. Momentum as ML^2/3 + ML^2×Lw0/2 (as u=Lw0) ........Sir pls reply.......
@padhai_karle_bhai
@padhai_karle_bhai 3 жыл бұрын
Did you got your answer?
@nikhilparikh2554
@nikhilparikh2554 3 жыл бұрын
Sir in this can't we do by linear momentum conservation ....we get v=u/3 ....and the from v=rw we get w as u/3l....???
@madihasiddiqui4540
@madihasiddiqui4540 3 жыл бұрын
no, because linear velocities of particles of the rod will be different and also hinge force is present preventing rods linear motion
@kunalverma6940
@kunalverma6940 6 жыл бұрын
Sir how is angular momentum conserved here since torque should not be zero..the rod should experience a torque
@rohanphutke4999
@rohanphutke4999 2 жыл бұрын
both small mass and rod are considered as system
@surajkumarbehera
@surajkumarbehera 5 жыл бұрын
sir, when will the links to download class notes after fundamental classes will be available ?. for now downloads are not available. I want to solve the problems again after class. and you strictly prohibited to take notes during class.
@physicsgalaxyworld
@physicsgalaxyworld 5 жыл бұрын
The app PG LIVE on which these videos are streamed is going to be re-launched after which notes will be available...
@surajkumarbehera
@surajkumarbehera 5 жыл бұрын
@@physicsgalaxyworld Sir please tell when will it be relaunched? And when will the test series ( premium features ) will come ?
@raghavs9241
@raghavs9241 6 жыл бұрын
Dear sir How initial angular momentum is M/2 u L . and also sir can you clarify about the axis of rotation in this example?
@physicsgalaxyworld
@physicsgalaxyworld 6 жыл бұрын
You need to first understand the basics of angular momentum as how it is calculated... the linear momentum of moving mass M/2 is Mu/2 and about the hinge of rod which is its AOR the angular momentum will be the product of linear momentum and the perpendicular separation of AOR from the line of motion so it is Mu/2 x L... we suggest you to follow our videos in proper sequence either on our mobile app 'Physics Galaxy' or through our website www.physicsgalaxy.com or our channel playlists... you missed the videos previous to this in sequence thats why you are facing this problem...
@raghavs9241
@raghavs9241 6 жыл бұрын
Thank you sir.... Got it now......
@NikhilKumar-ys9up
@NikhilKumar-ys9up 3 жыл бұрын
After collision, I considered mass of B and ball together (3M/2). Then it's their final angular momentum about A is (3M/2)(L)(required velocity), as it is in circular motion, and equate it to initial angular momentum. But my answer is different, please tell me where I am wrong sir?
@NikhilKumar-ys9up
@NikhilKumar-ys9up 3 жыл бұрын
Gotcha😅
@shivanshuverma5981
@shivanshuverma5981 7 жыл бұрын
hello sir! why did u say tht the angular momentum is conserved about point 'A'.?
@Pepe-jw7xw
@Pepe-jw7xw 3 жыл бұрын
Because if we take from any other point then we have to consider the hinge forces also and that will not result in conservation of momentum
@padhai_karle_bhai
@padhai_karle_bhai 3 жыл бұрын
@@Pepe-jw7xw but then why in initial momentum we took ball, ball ne to abhi hit bhi nahi kiya...
@ishan__001
@ishan__001 2 жыл бұрын
@@padhai_karle_bhai talking abt system sis
@padhai_karle_bhai
@padhai_karle_bhai 2 жыл бұрын
@@ishan__001 I'm a brother 😂😂
@ishan__001
@ishan__001 2 жыл бұрын
@@padhai_karle_bhai kitna rank aya brother mujhe dar lg rha h
@AshokKumar-qi4nr
@AshokKumar-qi4nr 7 жыл бұрын
Sir, here why can't we conserve linear momentum?
@aryanpratapsrivastava3757
@aryanpratapsrivastava3757 4 жыл бұрын
Impulsive force acts on it
@hotscandium6892
@hotscandium6892 4 жыл бұрын
Due to rotational motion the centrifugal force acts and hence net force is not zero but yes torque net is zero
@deuteriumtritium9700
@deuteriumtritium9700 5 жыл бұрын
Sir, as the rod moves up, won't there be torque due to gravitation force?
@kartikmessner2868
@kartikmessner2868 4 жыл бұрын
during collision we only have to consider impulsive forces..gravity is a non impusive force..thus torque due to it will also be non impulsive..thus we can neglect it.
@deuteriumtritium9700
@deuteriumtritium9700 4 жыл бұрын
@@kartikmessner2868 Thanks :)
@stealthinator16
@stealthinator16 4 жыл бұрын
The better answer is that we need to find angular velocity just after the impact. As the rod remains vertical, gravity will pass through the axis of rotation hence it's torque will be zero.
@SanjeevSingh-gx7bq
@SanjeevSingh-gx7bq 3 жыл бұрын
Sir this question came in 20 July mains paper
@manojwadhwa1856
@manojwadhwa1856 2 жыл бұрын
Jee advanced 2020 mai bhi similar ek question tha
@alexs6284
@alexs6284 6 жыл бұрын
where is torque of hinge force ???
@ASHWING.P
@ASHWING.P 2 ай бұрын
Aree yarr the axis is taken about the hinge
@nSackStyles
@nSackStyles 6 жыл бұрын
Since no external force acts, why can't we solve it using conservation of momentum?
@next_thing_to_god
@next_thing_to_god 7 ай бұрын
the rod is freking rotating bro
@divitgoyal5901
@divitgoyal5901 6 жыл бұрын
Sir can`t we just conserve linear momentum?And secondly why angular momentum only about point A is conserved and not everywhere?
@physicsgalaxyworld
@physicsgalaxyworld 6 жыл бұрын
Due to the impulse applied at the hinge linear momentum is not conserved in this case and about A only the torque due to force by impulse is zero so angular momentum can be conserved only about A in this case...
@divitgoyal5901
@divitgoyal5901 6 жыл бұрын
Physics Galaxy why about only A torque due to force by impulse is zero??
@repudirajasekhar3884
@repudirajasekhar3884 6 жыл бұрын
Sir...plz explain in detail... why angular momentum is conserved only at point A.... explain clearly sir... i have doubts about conservation of ang momentum
@nayannathani32
@nayannathani32 8 жыл бұрын
can we think of solving this question using energy conservation?
@physicsgalaxyworld
@physicsgalaxyworld 8 жыл бұрын
As the particle sticks to the rod after collision, here energy loss will occur due to inelastic collision... so energy cannot be conserved...
@gigachad6844
@gigachad6844 7 жыл бұрын
Sir, won't the mass after collision become 3M/2 ?
@physicsgalaxyworld
@physicsgalaxyworld 7 жыл бұрын
Here we are considering bodies independently...
@saicharanreddy1053
@saicharanreddy1053 7 жыл бұрын
particle hit acts force
@satadhi
@satadhi 9 жыл бұрын
hi! why did u say tht the angular momentum is conserverd about point 'A'. What about the others points ,?
@physicsgalaxyworld
@physicsgalaxyworld 9 жыл бұрын
Satadhi Halder When ball hits the rod, an impulse is imparted by hinge at A due to which at all other points there will be a torque of this force so angular momentum can only be conserved about point A.
@Rajatkumar-dm5uf
@Rajatkumar-dm5uf 8 жыл бұрын
but still sir torque would still be created about point of collision as there is force and l bieng teh perpendicular distance ?
@adarshchaturvedi3498
@adarshchaturvedi3498 8 жыл бұрын
Physics Galaxy sir please clarify this point again .. Why impulse is imparted only at hinge point and not other points .
@physicsgalaxyworld
@physicsgalaxyworld 8 жыл бұрын
Because only at this point any external force due to hinge is present... there is no other point on rod where a force can act...
@adarshchaturvedi3498
@adarshchaturvedi3498 8 жыл бұрын
Sir on this topic (conservation of angular momentum) I have some doubts. Sir when and where can I put it to u ..
I tried World's TOUGHEST Physics Exam Question
17:50
ZPhysics
Рет қаралды 89 М.
Officer Rabbit is so bad. He made Luffy deaf. #funny #supersiblings #comedy
00:18
Funny superhero siblings
Рет қаралды 19 МЛН
MY HEIGHT vs MrBEAST CREW 🙈📏
00:22
Celine Dept
Рет қаралды 70 МЛН
哈哈大家为了进去也是想尽办法!#火影忍者 #佐助 #家庭
00:33
火影忍者一家
Рет қаралды 129 МЛН
Solving one of the toughest Indian exam questions
21:12
Tibees
Рет қаралды 1,8 МЛН
Viral Video of a Man's Crazy Job Interview
16:02
Darryl Vega TV
Рет қаралды 1,4 МЛН
8.01x - Lect 24 - Rolling Motion, Gyroscopes, VERY NON-INTUITIVE
49:13
Lectures by Walter Lewin. They will make you ♥ Physics.
Рет қаралды 5 МЛН
You don't really understand physics
11:03
Ali the Dazzling
Рет қаралды 137 М.
LOSING WEIGHT & BEERBICEPS | Standup Comedy by Gautham Govindan
15:40
Gautham Govindan
Рет қаралды 311 М.
UCF Professor Richard Quinn accuses class of cheating [Original]
15:00
SubZeroCobra
Рет қаралды 15 МЛН
Gravity Visualized
9:58
apbiolghs
Рет қаралды 139 МЛН
Officer Rabbit is so bad. He made Luffy deaf. #funny #supersiblings #comedy
00:18
Funny superhero siblings
Рет қаралды 19 МЛН